Olá, Comunidade!

Vocês devem ter notado que o site ficou um período fora do ar (do dia 26 até o dia 30 de maio de 2024).

Consegui recuperar tudo, e ainda fiz um UPGRADE no servidor! Agora estamos em um servidor dedicado no BRASIL!
Isso vai fazer com que o acesso fique mais rápido (espero 🙏)

Já arrumei os principais bugs que aparecem em uma atualização!
Mas, se você encontrar alguma coisa diferente, que não funciona direito, me envie uma MP avisando que eu arranjo um tempo pra arrumar!

Vamos crescer essa comunidade juntos 🥰

Grande abraço a todos,
Prof. Caju

Maratonas de Matemátical Maratona Olímpica de Teoria dos Números

Moderador: [ Moderadores TTB ]

Avatar do usuário

Autor do Tópico
Ittalo25
5 - Mestre
Mensagens: 2349
Registrado em: 18 Nov 2013, 22:11
Última visita: 27-03-24
Agradeceu: 299 vezes
Agradeceram: 1401 vezes
Jan 2021 04 15:23

Re: l Maratona Olímpica de Teoria dos Números

Mensagem não lida por Ittalo25 »

Solução do problema 69

O polinômio característico dessa recorrência é [tex3]x^2 =1996x+1997 [/tex3] , o qual tem raízes [tex3]-1[/tex3] e [tex3]1997 [/tex3] .
Então:
[tex3]\begin{cases}
1=x\cdot (-1)^2 + y\cdot (1997)^2 \\
3993= x\cdot (-1)^3+y\cdot (1997)^3
\end{cases}[/tex3]
Somando as 2:
[tex3]\frac{3994}{(1997)^3+(1997)^2} = y[/tex3]
[tex3]\frac{2}{(1997)^2+(1997)} = y[/tex3]
[tex3]\frac{1}{1997 \cdot 999} = y[/tex3]
[tex3]-\frac{998}{ 999} = x[/tex3]

Assim:
[tex3]x_{1997} = -\frac{998}{999} \cdot (-1)^{1997} + \frac{(1997)^{1997}}{1997 \cdot 999}[/tex3]
[tex3]x_{1997} = \frac{998+(1997)^{1996}}{ 999}[/tex3]
[tex3]x_{1997} = 1+\frac{(1997)^{1996}-1}{ 999}[/tex3]
[tex3]x_{1997} = 1+\frac{((1997)^{499})^{4}-1}{ 999}[/tex3]
[tex3]x_{1997} = 1+\frac{((1997)^{499}-1)((1997)^{499}+1)((1997)^{998}+1)}{ 999}[/tex3]
[tex3]x_{1997} = 1+\frac{((1997)^{499}-1)(1998)(1997^{498}-1997^{497}+1997^{496}-1997^{495}+....-1997+1)((1997)^{998}+1)}{ 999}[/tex3]
[tex3]x_{1997} = 1+2\cdot ((1997)^{499}-1)(1997^{498}-1997^{497}+1997^{496}-1997^{495}+....-1997+1)((1997)^{998}+1)[/tex3]
[tex3]x_{1997} \equiv 1+2\cdot ((-1)^{499}-1)((-1)^{498}-(-1)^{497}+(-1)^{496}-(-1)^{495}+....+1+1)((-1)^{998}+1)\mod(3)[/tex3]
[tex3]x_{1997} \equiv 1+2\cdot (-2)\cdot (499)\cdot (2)\mod(3)[/tex3]
[tex3]x_{1997} \equiv 1-8\mod(3)[/tex3]
[tex3]\boxed{x_{1997} \equiv 2\mod(3)}[/tex3]

________________________________________________________________________________________________________

Problema 70
(Kosovo - 2013) Encontre todos os inteiros n tais que [tex3]\frac{n^2+n-27}{n-5}[/tex3] é um número inteiro.
Resposta

2,4,6,8

Editado pela última vez por Ittalo25 em 04 Jan 2021, 15:27, em um total de 2 vezes.
Ninguém pode ser perfeito, mas todos podem ser melhores. [\Bob Esponja]
Avatar do usuário

BotoCorDeRosa
sênior
Mensagens: 23
Registrado em: 15 Out 2020, 11:16
Última visita: 30-05-21
Jan 2021 04 22:06

Re: l Maratona Olímpica de Teoria dos Números

Mensagem não lida por BotoCorDeRosa »

Solução do problema 70

Usando o denominador:
[tex3]n - 5\equiv 0 \mod(n - 5)[/tex3]
[tex3]n\equiv 5 \mod(n - 5)[/tex3]
[tex3]n^2\equiv 5^2 \mod(n - 5)[/tex3]
[tex3]n^2 + n \equiv25 + 5\mod(n - 5)[/tex3]
[tex3]n^2 + n - 27 \equiv 30 - 27 \mod(n - 5)[/tex3]
[tex3]n^2 + n - 27 \equiv 3 \mod(n - 5)[/tex3]

Então, [tex3](n - 5)|3[/tex3] , divisores inteiros de 3: - 1, - 3, 1, 3

[tex3]n - 5 = - 1[/tex3]
[tex3]n = 5 - 1[/tex3]
[tex3]n = 4[/tex3]

[tex3]n - 5 = - 3[/tex3]
[tex3]n = 5 - 3[/tex3]
[tex3]n = 2[/tex3]

[tex3]n - 5 = 1[/tex3]
[tex3]n = 5 + 1[/tex3]
[tex3]n = 6[/tex3]

[tex3]n - 5 = 3[/tex3]
[tex3]n = 5 + 3[/tex3]
[tex3]n = 8[/tex3]

Problema 71
(Finlândia - 2001) Determine [tex3]n\in \mathbb{N}[/tex3] tais que [tex3]n^2 + 2[/tex3] divida [tex3]2 + 2001n[/tex3] .

Editado pela última vez por BotoCorDeRosa em 04 Jan 2021, 22:21, em um total de 1 vez.
Avatar do usuário

Autor do Tópico
Ittalo25
5 - Mestre
Mensagens: 2349
Registrado em: 18 Nov 2013, 22:11
Última visita: 27-03-24
Agradeceu: 299 vezes
Agradeceram: 1401 vezes
Jan 2021 05 00:25

Re: l Maratona Olímpica de Teoria dos Números

Mensagem não lida por Ittalo25 »

Solução do problema 71

[tex3]n^2+2 | 2+2001n [/tex3]
[tex3]n^2+2 | n\cdot (2+2001n) - 2001 \cdot (n^2+2) [/tex3]
[tex3]n^2+2 | 2n-4002[/tex3]
[tex3]n^2+2 | 2001\cdot (2n-4002) - 2\cdot (2+2001n)[/tex3]
[tex3]n^2+2 | 2\cdot (2001^2 +2)[/tex3]
[tex3]n^2+2 | 2\cdot 19 \cdot 83 \cdot 2539[/tex3]

[tex3]\begin{cases}
n^2+2 = 2\rightarrow \boxed{n = 0} \\
n^2+2 = 2\cdot 19 \rightarrow \boxed{n=6} \\
n^2+2 = 2\cdot 19 \cdot 83\rightarrow n = \sqrt
{3152} \\
n^2+2 = 2\cdot 19 \cdot 83 \cdot 2539 = \sqrt{8008004} \\
n^2+2 = 19\rightarrow n = \sqrt{17} \\
n^2+2 = 19 \cdot 83\rightarrow n=\sqrt{1575} \\
n^2+2 = 19\cdot 83 \cdot 2539\rightarrow \boxed{n = 2001} \\
n^2+2 = 83\rightarrow \boxed{n=9} \\
n^2+2 = 83 \cdot 2539\rightarrow n=210735 \\
n^2+2=2539\rightarrow n=\sqrt{2537} \\
\end{cases}[/tex3]
[tex3]\boxed{ n\in \{0,6,9,2001\}} [/tex3]

___________________________________________________________________________________________________________

Problema 72
(México - 1988) Sejam a e b inteiros positivos. Se 19 divide [tex3]11a+2b [/tex3] , então prove que 19 também divide [tex3]18a+5b [/tex3]
Ninguém pode ser perfeito, mas todos podem ser melhores. [\Bob Esponja]
Avatar do usuário

BotoCorDeRosa
sênior
Mensagens: 23
Registrado em: 15 Out 2020, 11:16
Última visita: 30-05-21
Jan 2021 05 12:11

Re: l Maratona Olímpica de Teoria dos Números

Mensagem não lida por BotoCorDeRosa »

Solução do problema 72

[tex3]11a + 2b \equiv 0 \mod(19)[/tex3]
[tex3]11a \equiv - 2b \mod(19)[/tex3]

[tex3]11 \equiv (- 8) \mod(19)[/tex3]
[tex3]11a \equiv (- 8)a \mod(19)[/tex3]

Então:
[tex3](- 8)a \equiv -2b \mod(19)[/tex3]
[tex3]2 \times(4a) \equiv 2 \times (b) \mod(19)[/tex3] , como [tex3]19 \nmid 2[/tex3] :
[tex3]4a \equiv b \mod(19)[/tex3]

[tex3]18a + 5b \equiv x \mod(19)[/tex3]
[tex3]18a + 5(4a) \equiv x \mod(19)[/tex3]
[tex3]18a + 20a \equiv x \mod(19)[/tex3]
[tex3]38a \equiv x \mod(19)[/tex3]
[tex3]19(2a) \equiv x \mod(19)\therefore \boxed {x=0}[/tex3]

Problema 73
(Estônia - 2002) Um número natural de 10 algarismos distintos é dito mágico se ele for múltiplo de 99999. Quantos números mágicos não começados por zero existem?
Avatar do usuário

Autor do Tópico
Ittalo25
5 - Mestre
Mensagens: 2349
Registrado em: 18 Nov 2013, 22:11
Última visita: 27-03-24
Agradeceu: 299 vezes
Agradeceram: 1401 vezes
Jan 2021 06 01:23

Re: l Maratona Olímpica de Teoria dos Números

Mensagem não lida por Ittalo25 »

Solução do problema 73

Seja o número mágico: [tex3]\overline{x_1x_2x_3x_4x_5....x_{10}}[/tex3] , então:

[tex3]\overline{x_1x_2x_3x_4x_5....x_{10}} = 10^5\cdot \overline{x_1x_2x_3x_4x_5}+ \overline{x_6x_7x_8x_9x_{10}}[/tex3]
[tex3]\overline{x_1x_2x_3x_4x_5....x_{10}} = 99999\cdot \overline{{x_1x_2x_3x_4x_5}}+ \overline{{x_1x_2x_3x_4x_5}}+ \overline{{x_6x_7x_8x_9x_{10}}}[/tex3]

Então 99999 divide [tex3]\overline{{x_1x_2x_3x_4x_5}}+ \overline{{x_6x_7x_8x_9x_{10}}}[/tex3]
Mas [tex3]x_i\neq x_j [/tex3] para [tex3]i\neq j[/tex3] (A questão diz isso)
Consequência disso é que [tex3]\overline{{x_1x_2x_3x_4x_5}}<99999[/tex3] e também [tex3]\overline{{x_6x_7x_8x_9x_{10}}}<99999[/tex3] .
Sendo assim: [tex3]\overline{{x_1x_2x_3x_4x_5}}+ \overline{{x_6x_7x_8x_9x_{10}}} = 99999[/tex3]
Portanto: [tex3]x_1+x_6 = x_2+x_7=x_3+x_8=x_4+x_9 = x_5+x_{10}=9 [/tex3]

Para [tex3]x_1+x_6 [/tex3] : [tex3](x_1\neq 0)[/tex3]
[tex3]\begin{cases}
9+0 \\
8+1 \\
7+2 \\
6+3 \\
5+4\\
4+5 \\
3+6 \\
2+7 \\
1+8
\end{cases}[/tex3]
9 possibilidades.

Para [tex3]x_2+x_7 [/tex3] :
Aqui vai adicionar 1 caso já que não existe restrição para [tex3]x_2,x_7\neq 0[/tex3]
Vai retirar 1 caso que já foi escolhido anteriormente e também o caso simétrico. Por exemplo: Se [tex3]x_1 = 8 [/tex3] , [tex3]x_6=1 [/tex3] , não é possível ter [tex3]x_2=1 [/tex3] e [tex3]x_7=8 [/tex3] nem [tex3]x_2=8 [/tex3] e [tex3]x_7=1 [/tex3]
8 possibilidades

Para [tex3]x_3+x_8 [/tex3] :
Aqui retira-se o caso escolhido anteriormente e seu simétrico.
6 possibilidades

Para [tex3]x_4+x_9 [/tex3] :
Aqui retira-se o caso escolhido anteriormente e seu simétrico.
4 possibilidades

Para [tex3]x_5+x_{10} [/tex3] :
Aqui retira-se o caso escolhido anteriormente e seu simétrico.
2 possibilidades

Então pelo princípio multiplicativo, o número total de números mágicos é: [tex3]\boxed{9\cdot 8\cdot 6\cdot 4\cdot 2=3456} [/tex3]

____________________________________________________________________________________________________________

Problema 74
(México - 1997) Encontre todos os primos p tais que [tex3]8p^{4}-3003[/tex3] é um número primo positivo.
Resposta

p=5
Ninguém pode ser perfeito, mas todos podem ser melhores. [\Bob Esponja]

Deleted User 25040
6 - Doutor
Última visita: 31-12-69
Mar 2021 11 13:58

Re: l Maratona Olímpica de Teoria dos Números

Mensagem não lida por Deleted User 25040 »

Solução Problema 74
devido ao expoente ser [tex3]4 = 5-1[/tex3] talvez seja uma boa ideia olhar módulo 5 para poder usar o teorema de euler, fazendo isso temos o seguinte.
[tex3]8p^4-3003\equiv3p^4-3\equiv3(p^4-1)(\mod5)[/tex3]
se [tex3](p,5)=1[/tex3] o número do enunciado sera divisível por 5, vamos testar isso então.
[tex3]8p^4-3003=5[/tex3]
[tex3]8p^4=3008[/tex3]
[tex3]p^4=376=2^3\cdot47[/tex3] então [tex3]2|p^4[/tex3] como 2 é primo e divide um produto [tex3]2|p\implies p = 2[/tex3] mas
[tex3]2^4\neq2^3\cdot47[/tex3]
como [tex3](p, 5) = 1 \text{ ou }5[/tex3] dependendo se [tex3]5\nmid p[/tex3] ou [tex3]5|p [/tex3] respectivamente
só falta testar [tex3]5|p[/tex3] e pelo fato de p ser primo, p = 5
[tex3]8\cdot5^4-3003=1997[/tex3] que é primo, lembre-se que só precisamos dividir 1997 por primos até [tex3]\sqrt{1997}[/tex3] devido ao seguinte teorema.
Resposta

Se n inteiro positivo não é primo, então n possui, necessariamente, um fator primo menor do que ou igual a [tex3]\sqrt{n}[/tex3]
demonstração: [tex3]n=n_1n_2[/tex3] , suponha sem perda de generalidade [tex3]n_1\leq n_2[/tex3] , então [tex3]n_1\leq \sqrt{n}[/tex3] caso contrário [tex3]n = n_1n_2>\sqrt{n}\sqrt{n}=n[/tex3] , absurdo, como [tex3]n_1[/tex3] deve ter um fator primo e se [tex3]p[/tex3] for fator primo de [tex3]n_1[/tex3] tbm sera fator primo de [tex3]n[/tex3] , a prova está completa.

Problema 75
(APMO - 1997) Encontre um [tex3]n[/tex3] no conjunto [tex3]\{100, 101, ..., 1997\}[/tex3] tal que [tex3]n[/tex3] divide [tex3]2^n+2[/tex3]
Editado pela última vez por Deleted User 25040 em 11 Mar 2021, 13:59, em um total de 1 vez.

iammaribrg
2 - Nerd
Mensagens: 230
Registrado em: 11 Mai 2020, 18:14
Última visita: 21-05-23
Abr 2021 06 00:46

Re: l Maratona Olímpica de Teoria dos Números

Mensagem não lida por iammaribrg »

Solução Problema 75

Temos que encontrar n = 2 [tex3]\cdot [/tex3] p [tex3]\cdot [/tex3] q, p, q primos ímpares distintos. Devemos ter [tex3]2^{2pq-1}[/tex3] −1 + 1 ≡ 0 (mod p) ⇐⇒[tex3]2^{2q-1}[/tex3] ≡ −1 (mod p). Veja que [tex3](2^{\frac{p-1}{2}})^{2}[/tex3] ≡ 1 (mod p) ⇐⇒ (p−1)/2 ≡ ±1 (mod p). Se [tex3]2^{\frac{p-1}{2}}[/tex3] ≡ −1 (mod p), podemos tentar 2q − 1 = (p − 1)/2 ⇐⇒ p = 4q − 1. Analogamente, 2 [tex3]^{2p-1}[/tex3] ≡ −1 (mod q) ⇐⇒ 2 [tex3]^{8q-3}[/tex3] ≡ −1 (mod q) ⇐⇒ 2 [tex3]^{5}[/tex3] ≡ −1 (mod q) ⇐⇒ q = 3 ou q = 11. No primeiro caso, p = 11 e n = 66; no segundo caso, p = 43 e n = 946. De fato, uma busca no computador mostra que n = 946 é a única possibilidade

Problema 76
(Marrocos - 2021) Sejam a e b números reais positivos, tais que: [tex3]\begin{cases}
ab=2 \\
\frac{a}{a+b^2}+\frac{b}{b+a^2} = \frac{7}{8}
\end{cases}[/tex3] , encontre o valor de [tex3]a^6+b^6[/tex3]
Editado pela última vez por Ittalo25 em 07 Abr 2021, 20:08, em um total de 4 vezes.
O fogo arderá continuamente sobre o altar; não se apagará.
Levítico 6:13

Deleted User 25040
6 - Doutor
Última visita: 31-12-69
Jul 2021 18 21:12

Re: l Maratona Olímpica de Teoria dos Números

Mensagem não lida por Deleted User 25040 »

Solução do problema 76.
[tex3]\frac{a}{a+b^2}+\frac{b}{b+a^2} = \frac{7}{8}\implies8(a(b+a^2)+b(a+b^2))=7(a+b^2)(b+a^2)\iff a^3 + 9 a b + b^3 = 7 a^2 b^2[/tex3]
usando [tex3]ab=2[/tex3]
[tex3]a^3+b^3=7\cdot4-9\cdot2=10[/tex3]
elevando os dois lados ao quadrado obtemos a seguinte igualdade:
[tex3]a^6+2a^3b^3+b^6=100[/tex3] usando novamente [tex3]ab = 2[/tex3]
[tex3]a^6+b^6=84[/tex3]

Problema 77.
(All Russia Mathematics Olympiad - 1995)
sejam m e n inteiros positivos tais que
[tex3]mmc(m, n) + mdc(m, n) = m+n[/tex3]
mostre que um dos números é divisível pelo outro.
Avatar do usuário

Autor do Tópico
Ittalo25
5 - Mestre
Mensagens: 2349
Registrado em: 18 Nov 2013, 22:11
Última visita: 27-03-24
Agradeceu: 299 vezes
Agradeceram: 1401 vezes
Ago 2021 05 14:22

Re: l Maratona Olímpica de Teoria dos Números

Mensagem não lida por Ittalo25 »

Solução do problema 77

é bem sabido que [tex3]mdc(m,n) \cdot mmc(m,n) = mn [/tex3] , portanto:
[tex3]mmc(m, n) + mdc(m, n) = m+n[/tex3]
[tex3]mmc(m, n) +\frac{mn}{mmc(m,n)} = m+n[/tex3]
[tex3]mmc^2(m, n)-(m+n)\cdot mmc(m,n) +mn = 0[/tex3]
[tex3]mmc(m,n)=\frac{m+n\pm \sqrt{(m+n)^2-4 mn}}{2}[/tex3]
[tex3]mmc(m,n)=\frac{m+n\pm |m-n|}{2}[/tex3]
Sendo assim: [tex3]\boxed{mmc(m,n)\in \{m,n\}}[/tex3]

---------------------------------------------------------------------------------------------------------------------------------------------

Problema 78
(Polônia - 2018) Encontre os trios de inteiros (x,y,z) que são soluções do sistema: [tex3]\begin{cases}
x-yz=1 \\
xz+y=2
\end{cases}[/tex3]
Resposta

(1,0,2) , (1,2,0)
Ninguém pode ser perfeito, mas todos podem ser melhores. [\Bob Esponja]
Avatar do usuário

AnthonyC
4 - Sabe Tudo
Mensagens: 964
Registrado em: 09 Fev 2018, 19:43
Última visita: 21-02-24
Agradeceu: 1 vez
Agradeceram: 2 vezes
Ago 2021 06 17:23

Re: l Maratona Olímpica de Teoria dos Números

Mensagem não lida por AnthonyC »

Solução do Problema 78:
[tex3]\begin{cases}
x-yz=1 \\
xz+y=2
\end{cases}[/tex3]
[tex3]\begin{cases}
x^2-xyz=x \\
xyz+y^2=2y
\end{cases}[/tex3]
Somando ambas as equações:
[tex3]x^2+y^2= x+2y[/tex3]
[tex3]x^2-x+y^2-2y=0[/tex3]
[tex3]x^2-x+{1\over4}+y^2-2y+1={5\over4}[/tex3]
[tex3]\(x-{1\over2}\)^2+(y-1)^2={5\over4}[/tex3]
[tex3]\(x-{1\over2}\)^2={5\over4}-(y-1)^2[/tex3]
[tex3]x-{1\over2}=\pm\sqrt{{5\over4}-(y-1)^2}[/tex3]
[tex3]x={1\over2}\pm\sqrt{{5\over4}-(y-1)^2}~~~~~~~~~~(I)[/tex3]
Para que haja soluções reais, temos que [tex3]{5\over4}-(y-1)^2\geq0\implies {\sqrt{5}\over2}\geq|y-1|\implies1-{\sqrt{5}\over2}\leq y\leq{1+\sqrt{5}\over2}[/tex3] . Como estamos interessados em [tex3]y\in\mathbb{Z}[/tex3] , temos:
[tex3]y\in\[1-{\sqrt{5}\over2},{1+\sqrt{5}\over2}\]\cap\mathbb{Z}[/tex3]
[tex3]y\in\left\{0,1,2\right\}[/tex3]
Subsituindo este três valores em (I), temos:
  • [tex3]y=0\implies x=0 \text{ ou } 1[/tex3]
  • [tex3]y=1\implies x={1\over2}\pm {\sqrt5\over2} [/tex3]
  • [tex3]y=2\implies x=0 \text{ ou } 1[/tex3]
Considerando apenas soluções inteiras, temos [tex3](x,y)\in\{(0,0),(1,0),(0,2),(1,2)\}[/tex3] . Testando estas opções nas equações originais:
  • [tex3](x,y)=(0,0)\implies\begin{cases}
    0=1 \\
    0=2
    \end{cases}~~(\text{impossível})[/tex3]
  • [tex3](x,y)=(1,0)\implies\begin{cases}
    1=1 \\
    z=2
    \end{cases}[/tex3]
  • [tex3](x,y)=(0,2)\implies\begin{cases}
    -2z=1\implies z=-{1\over2} \\
    2=2
    \end{cases}~~(\text{não serve, pois } z\in\mathbb{Z} )[/tex3]
  • [tex3](x,y)=(1,2)\implies\begin{cases}
    1-2z=1\implies z=0 \\
    z+2=2\implies z=0
    \end{cases}[/tex3]
Assim, temos o seguinte conjunto solução:
[tex3](x,y,z)=\{(1,0,2),(1,2,0)\}[/tex3]
Problema 79
(Olímpiada Aberta de Matemática 239-2011)
Sejam [tex3]a,b,c\in\mathbb{N}[/tex3] , satisfazendo [tex3]\begin{cases}
a+b=b(a-c) \\
c+1=p^2, ~~\text{para um primo } p \text{ qualquer}
\end{cases}[/tex3] .
Mostre que [tex3]a+b[/tex3] ou [tex3]ab[/tex3] é um quadrado perfeito.

[tex3]\color{YellowOrange}\textbf{Não importa o quanto se esforce ou evolua, você sempre estará abaixo do Sol}[/tex3]
[tex3]\textbf{Escanor}[/tex3]
Responder
  • Tópicos Semelhantes
    Respostas
    Exibições
    Última mensagem

Voltar para “Maratonas de Matemática”